Combine two different size matrix in state equations

  • Thread starter Thread starter khedira
  • Start date Start date
  • Tags Tags
    Matrix State
Click For Summary
The discussion revolves around combining two sets of state equations with matrices of different sizes. The user initially struggles with adding a 2x2 matrix A1 and a 1x1 matrix A2 due to their size difference. They later realize that A2 can be treated as a scalar, allowing for the addition to be performed correctly. A suggestion is made to use A2 multiplied by an identity matrix to facilitate the combination. The conversation highlights the importance of understanding matrix dimensions in state equations.
khedira
Messages
11
Reaction score
0
i have two sets of state equations:

1) x(dot) = (2x2)x + (2x1)u
y = (1x2)x

2) x(dot) = (0)x + (1x1)u
y = (1x1)x

given the above, since A, B and C are of different sizes, how can i add the A from 1) and 2) to get a combined A? Is that possible? Please advise. Thank you.
 
Physics news on Phys.org
what are A, B & C? can you explain better?

also what are x,y & u are these 2x1 column vectors etc.
 
lanedance said:
what are A, B & C? can you explain better?

also what are x,y & u are these 2x1 column vectors etc.

Hi lanedance,

Thank you for your reply. I confused myself and is able to get the answer already.

The problem was that i have this A1 2x2 matrix, e.g. [1 2; 3 4] and i have another A2 1x1 matrix [5]. I wish to add them together, i.e. A = A1 + A2. At first i thought that the size different between A1 and A2 make it impossible to directly add them up. Then i realize that since A2 is in scalar form, A = A1 + A2 = [6 7; 8 9].

Thank you for your reply anyway.
 
yeah that doesn't seem quite right, maybe if its A2*I , where I is the identity matrix it works
 
Question: A clock's minute hand has length 4 and its hour hand has length 3. What is the distance between the tips at the moment when it is increasing most rapidly?(Putnam Exam Question) Answer: Making assumption that both the hands moves at constant angular velocities, the answer is ## \sqrt{7} .## But don't you think this assumption is somewhat doubtful and wrong?

Similar threads

Replies
2
Views
2K
Replies
2
Views
1K
Replies
5
Views
2K
  • · Replies 10 ·
Replies
10
Views
2K
Replies
3
Views
2K
Replies
3
Views
1K
  • · Replies 4 ·
Replies
4
Views
3K
Replies
6
Views
2K
  • · Replies 4 ·
Replies
4
Views
2K
Replies
3
Views
2K